成績を上げるためには自宅学習!

数学の解説動画の作成開始しました。
チャンネル登録お願いいたします。

動画ページへ

外積を利用した2つのベクトルに垂直なベクトルの求め方とは?

外積とは 数学IAIIB
スポンサーリンク

1つのベクトルの成分に0がない場合

さっきは成分に0が含まれていたから簡単にできましたけど,そうじゃない場合はどうするんですか?

ヒロ
ヒロ

次の問題で解説していくよ。

2018年 慶應義塾大・薬学部座標空間に4点 $\mathrm{O}(0,0,0)$,$\mathrm{A}(3,-2,-1)$,$\mathrm{B}(1,1,1)$,$\mathrm{C}(-1,4,2)$ がある。
$\Vec{OA}$,$\Vec{BC}$のどちらにも垂直で長さが $3\sqrt{3}$ であるベクトルを成分で表すと,$\myhako$ である。

プリントを次のリンクからダウンロードできます。
ヒロ
ヒロ

まず,$\Vec{OA},\Vec{BC}$ の成分を求めよう。

それは任せて下さい!

\begin{align*}
\Vec{OA}=(3,-2,-1),~\Vec{BC}=(-2,3,1)
\end{align*}
ヒロ
ヒロ

外積を利用して2つのベクトルに垂直なベクトルを求める方法をマスターしよう。通常,高校では習わないけど,知っておくと得・・・というか知らないと損なので教えておくよ。

ヒロ
ヒロ

まず,$\vec{a}=(a_1,~a_2,~a_3),~\vec{b}=(b_1,~b_2,~b_3)$とすると,
$\vec{a}$と$\vec{b}$ の外積は次のようになる。

\begin{align*}
\vec{a}\times\vec{b}=(a_2b_3-a_3b_2,~a_3b_1-a_1b_3,~a_1b_2-a_2b_1)
\end{align*}

これを覚えるんですか・・・?

ヒロ
ヒロ

そのまま覚えてって言っても,覚えにくいと思うので,次のように覚えよう。

ヒロ
ヒロ

まず,$\vec{a}$ の成分を $x$ 成分,$y$ 成分,$z$ 成分,$x$ 成分の順に並べて,その下に $\vec{b}$ の成分を $\vec{a}$ と同じ順に並べよう。あとはクロスに掛けて引く。成分は①,②,③の順に $x,y,z$ となるよ。

外積を求める計算方法
ヒロ
ヒロ

①,②,③は次の計算規則に従って計算しよう。

外積を求める計算方法の説明
ヒロ
ヒロ

とりあえず,このルールで $\Vec{OA}\times\Vec{BC}$ を求めてみよう。

やってみます!

外積を求める具体的な計算
\begin{align*}
①&=-2\times1-(-1)\times3 \\[4pt]&=-2+3 \\[4pt]&=1 \\[4pt]②&=-1\times(-2)-3\times1 \\[4pt]&=2-3 \\[4pt]&=-1 \\[4pt]③&=3\times3-(-2)\times(-2) \\[4pt]&=9-4 \\[4pt]&=5
\end{align*}
よって,$\Vec{OA}\times\Vec{BC}=(1,-1,5)$

出来ました!

ヒロ
ヒロ

じゃあ, $\Vec{OA}\times\Vec{BC}$ と $\Vec{OA},~\Vec{BC}$ の内積を計算してみて?

はい!

\begin{align*}
(\Vec{OA}\times\Vec{BC})\Cdota\Vec{OA}&=(1,-1,5)\Cdota(3,-2,-1) \\[4pt]&=3+2-5 \\[4pt]&=0 \\[4pt](\Vec{OA}\times\Vec{BC})\Cdota\Vec{BC}&=(1,-1,5)\Cdota(-2,3,1) \\[4pt]&=-2-3+5 \\[4pt]&=0
\end{align*}

両方とも内積が0になりました。

ということは,外積で求めたベクトルは,元の2つのベクトルと垂直なんですね。

ヒロ
ヒロ

そういうこと。これで1つの方向は求まったね。

1つの方向?もう1つあるんですか?

ヒロ
ヒロ

逆ベクトルがあるよね?こういう問題では注意しないとね。

なるほど・・・忘れてしまいますね。

ヒロ
ヒロ

次は大きさの調整だね。$\Vec{OA}\times\Vec{BC}$ の大きさを求めてみて?

\begin{align*}
\abs{\Vec{OA}\times\Vec{BC}}&=\sqrt{1^2+(-1)^2+5^2} \\[4pt]&=\sqrt{27} \\[4pt]&=3\sqrt{3}
\end{align*}

え・・・

大きさは $3\sqrt{3}$ です。

ヒロ
ヒロ

そうだね。偶然にも調整せずに済んだね。ってことで,答えは $\pm(1,-1,5)$ だね。

タイトルとURLをコピーしました